LSAT and Law School Admissions Forum

Get expert LSAT preparation and law school admissions advice from PowerScore Test Preparation.

 Administrator
PowerScore Staff
  • PowerScore Staff
  • Posts: 8916
  • Joined: Feb 02, 2011
|
#38401
Complete Question Explanation
(The complete setup for this game can be found here: lsat/viewtopic.php?t=15027)

The correct answer choice is (A)

This question establishes that W (along with V) is positioned earlier than R, and also that R is positioned earlier than T. Since Q must also be earlier than R, the following sequencing chain controls the variable placement in this question:
PT76_O15 LG Explanations_Game #1_#5_diagram 1.png
This sequence alone, even in the absence of a basic linear diagram, helps eliminate answer choices (B) and (E). Additionally, answer choice (C) can also be eliminated relatively quickly, because the VW Block must be placed earlier than P. As a result, only V or W can be second.

To differentiate between answer choices (A) and (D), it is sufficient to set one of them up: if your setup complies with all the rules, then that answer choice could be true and would be correct. If not, the remaining answer choice would be correct.

Answer choice (A) could be true, because Q could be 1 without violating any of the rules of the game:
PT76_O15 LG Explanations_Game #1_#5_diagram 2.png


By contrast, answer choice (D) cannot be true: If T is 6, that would force R to be 5. This is because P is 4, and at least three variables must be earlier than R (V, W, and Q). We are left with no choice but to place S last, in direct violation of the rotating TS Not-Block:
PT76_O15 LG Explanations_Game #1_#5_diagram 3.png
Therefore, answer choice (A) is correct.

Differentiating between answer choices (A) and (D) required staying mindful of the separation requirement conveyed by the first rule. Although Not Blocks are generally not sufficiently restrictive to use at the initial stages of your local setup, they can be critical in differentiating between two contenders in Could Be True questions, where both contenders satisfy all of the remaining rules.
You do not have the required permissions to view the files attached to this post.

Get the most out of your LSAT Prep Plus subscription.

Analyze and track your performance with our Testing and Analytics Package.